Difference between revisions of "2002 AIME I Problems/Problem 4"

(Solution)
(Solution)
Line 17: Line 17:
 
<math>\frac{29t}{t+1} = m</math>
 
<math>\frac{29t}{t+1} = m</math>
  
Since n is an integer, <math>t+1 = 29</math>, or <math>t=28</math>. It quickly follows that <math>n=29(28)</math> and <math>m=28</math>, so <math>m+n = 30(28) = \fbox{840}</math>.
+
Since m is an integer, <math>t+1 = 29</math>, or <math>t=28</math>. It quickly follows that <math>n=29(28)</math> and <math>m=28</math>, so <math>m+n = 30(28) = \fbox{840}</math>.
  
 
*If <math>m=29t</math>, a similar argument to the one above implies <math>m=29(28)</math> and <math>m=28</math>, which implies <math>m>n</math>, which is impossible since <math>n-m>0</math>.
 
*If <math>m=29t</math>, a similar argument to the one above implies <math>m=29(28)</math> and <math>m=28</math>, which implies <math>m>n</math>, which is impossible since <math>n-m>0</math>.

Revision as of 15:40, 5 March 2011

Problem

Consider the sequence defined by $a_k =\dfrac{1}{k^2+k}$ for $k\geq 1$. Given that $a_m+a_{m+1}+\cdots+a_{n-1}=\dfrac{1}{29}$, for positive integers $m$ and $n$ with $m<n$, find $m+n$.

Solution

$\dfrac{1}{k^2+k}=\dfrac{1}{k(k+1)}=\dfrac{1}{k}-\dfrac{1}{k+1}$. Thus,

$a_m+a_{m+1}++\cdots +a_{n-1}=\dfrac{1}{m}-\dfrac{1}{m+1}+\dfrac{1}{m+1}-\dfrac{1}{m+2}+\cdots +\dfrac{1}{n-1}-\dfrac{1}{n}=\dfrac{1}{m}-\dfrac{1}{n}$

Which is

$\dfrac{n-m}{mn}=\dfrac{1}{29}$

Since we need a 29 in the denominator, let $n=29t$.* Substituting,

$29t-m=mt$

$\frac{29t}{t+1} = m$

Since m is an integer, $t+1 = 29$, or $t=28$. It quickly follows that $n=29(28)$ and $m=28$, so $m+n = 30(28) = \fbox{840}$.

  • If $m=29t$, a similar argument to the one above implies $m=29(28)$ and $m=28$, which implies $m>n$, which is impossible since $n-m>0$.

See also

2002 AIME I (ProblemsAnswer KeyResources)
Preceded by
Problem 3
Followed by
Problem 5
1 2 3 4 5 6 7 8 9 10 11 12 13 14 15
All AIME Problems and Solutions